Difference between revisions of "Schwarzschild black hole"

From Universe in Problems
Jump to: navigation, search
(Problem 18.)
 
(35 intermediate revisions by 2 users not shown)
Line 1: Line 1:
[[Category:Black Holes]]
+
[[Category:Black Holes|2]]
  
The spherically symmetric solution of Einstein's equations in vacuum for the spacetime metric has the form \cite{Schw}
+
The spherically symmetric solution of Einstein's equations in vacuum for the spacetime metric has the form$^{*}$
 
\begin{align}\label{Schw}
 
\begin{align}\label{Schw}
 
ds^{2}=h(r)\,dt^2-h^{-1}(r)\,dr^2-r^2 d\Omega^{2},
 
ds^{2}=h(r)\,dt^2-h^{-1}(r)\,dr^2-r^2 d\Omega^{2},
 
&\qquad\mbox{where}\quad
 
&\qquad\mbox{where}\quad
 
h(r)=1-\frac{r_g}{r};\quad r_{g}=\frac{2GM}{c^{2}};\\
 
h(r)=1-\frac{r_g}{r};\quad r_{g}=\frac{2GM}{c^{2}};\\
d\Omega^{2}=d\theta^{2}+\sin^{2}\theta\, d\varphi^{2}&\;\text{-- metric of unit sphere.}\nonumber
+
d\Omega^{2}=d\theta^{2}+\sin^{2}\theta\, d\varphi^{2}&\;\text{is the metric of unit sphere.}\nonumber
 
\end{align}
 
\end{align}
The Birkhoff's theorem (1923) \cite{Birkhoff,Jebsen} states, that this solution is unique up to coordinate transformations. The quantity $r_g$ is called the Schwarzschild radius, or gravitational radius, $M$ is the mass of the central body or black hole.
+
The Birkhoff's theorem$^{**}$ (1923) states, that this solution is unique up to coordinate transformations. The quantity $r_g$ is called the Schwarzschild radius, or gravitational radius, $M$ is the mass of the central body or black hole.
 +
 
 +
$^{*}$ K. Schwarzschild, On the gravitational field of a mass point according to Einstein's theory, ''Sitzungsber. Preuss. Akad. Wiss. Phys. Math. Kl.'',  p.189 (1916) (there's a translation of the original paper at [http://arxiv.org/abs/physics/9905030v1 arXiv:physics/9905030v1]; please disregard the abstract/foreword, which is incorrect).
 +
 
 +
$^{**}$ G.D. Birkhoff, Relativity and Modern Physics, p.253, Harvard University Press, Cambridge (1923);
 +
J.T. Jebsen, "Ark. Mat. Ast. Fys." (Stockholm) 15, nr.18 (1921), see also [http://arxiv.org/abs/physics/0508163 arXiv:physics/0508163v2].
 +
 
 +
__TOC__
  
 
==Simple problems==
 
==Simple problems==
 
<div id="BlackHole15"></div>
 
<div id="BlackHole15"></div>
=== Problem 1. ===
+
=== Problem 1: local time ===
 
Find the interval of local time (proper time of stationary observer) at a point $(r,\theta,\varphi)$ in terms of coordinate time $t$, and show that $t$ is the proper time of an observer at infinity. What happens when $r\to r_{g}$?
 
Find the interval of local time (proper time of stationary observer) at a point $(r,\theta,\varphi)$ in terms of coordinate time $t$, and show that $t$ is the proper time of an observer at infinity. What happens when $r\to r_{g}$?
  
Line 20: Line 27:
 
     <p style="text-align: left;">
 
     <p style="text-align: left;">
 
The proper time of the stationary observer is $d\tau=ds|_{dr=d\theta=d\varphi=0}$:
 
The proper time of the stationary observer is $d\tau=ds|_{dr=d\theta=d\varphi=0}$:
\[d\tau=\sqrt{g_{00}}dt=\sqrt{1-\frac{r_{g}}{r}}dt.\]
+
\[d\tau=\sqrt{g_{00}}\;dt=\sqrt{1-\frac{r_{g}}{r}}\;dt.\]
 
At $r\to \infty$ it coincides with $dt$, so the coordinate time $t$ can be interpreted as the proper time of a "remote" observer. At $r\to r_g$ the local time flows slower and asymptotically stops. If one of two twins were to live some time at $r\approx r_g$, he will return to his remote twin having aged less (thought he might have acquired some grey hair due to constant fear of tumbling over the horizon). </p>
 
At $r\to \infty$ it coincides with $dt$, so the coordinate time $t$ can be interpreted as the proper time of a "remote" observer. At $r\to r_g$ the local time flows slower and asymptotically stops. If one of two twins were to live some time at $r\approx r_g$, he will return to his remote twin having aged less (thought he might have acquired some grey hair due to constant fear of tumbling over the horizon). </p>
 
   </div>
 
   </div>
Line 27: Line 34:
 
<div id="BlackHole16"></div>
 
<div id="BlackHole16"></div>
  
=== Problem 2. ===
+
=== Problem 2: measuring distances ===
 
What is the physical distance between two points with coordinates $(r_{1},\theta,\varphi)$ and $(r_{2},\theta,\varphi)$? Between $(r,\theta,\varphi_{1})$ and $(r,\theta,\varphi_{2})$? How do these distances behave in the limit $r_{1},r\to r_{g}$?
 
What is the physical distance between two points with coordinates $(r_{1},\theta,\varphi)$ and $(r_{2},\theta,\varphi)$? Between $(r,\theta,\varphi_{1})$ and $(r,\theta,\varphi_{2})$? How do these distances behave in the limit $r_{1},r\to r_{g}$?
 
<div class="NavFrame collapsed">
 
<div class="NavFrame collapsed">
Line 43: Line 50:
 
<div id="BlackHole17"></div>
 
<div id="BlackHole17"></div>
  
=== Problem 3. ===
+
=== Problem 3: the inner region ===
 
What would be the answers to the previous two questions for $r<r_g$ and why*? Why the Schwarzschild metric cannot be imagined as a system of "welded" rigid rods in $r<r_g$, as it can be in the external region?
 
What would be the answers to the previous two questions for $r<r_g$ and why*? Why the Schwarzschild metric cannot be imagined as a system of "welded" rigid rods in $r<r_g$, as it can be in the external region?
 
<div class="NavFrame collapsed">
 
<div class="NavFrame collapsed">
Line 49: Line 56:
 
   <div style="width:100%;" class="NavContent">
 
   <div style="width:100%;" class="NavContent">
 
     <p style="text-align: left;">
 
     <p style="text-align: left;">
''This is question is given not to be answered but to make think on the answer. Correct questions and correct answers can be given in terms of a proper coordinate frame, that is regular both in $r>r_g$ and in $r<r_g$. Still, one can say something meaningful as is.''</p>
+
''This question is given not to be answered but to make one think of the answer. Correct questions and correct answers can be given in terms of a proper coordinate frame, which is regular both in $r>r_g$ and in $r<r_g$. Still, one can say something meaningful as is.''</p>
  
 
<p style="text-align: left;">
 
<p style="text-align: left;">
Line 80: Line 87:
 
<div id="BlackHole18"></div>
 
<div id="BlackHole18"></div>
  
=== Problem 4. ===
+
=== Problem 4: acceleration ===
 
Calculate the acceleration of a test particle with zero velocity.
 
Calculate the acceleration of a test particle with zero velocity.
 
<div class="NavFrame collapsed">
 
<div class="NavFrame collapsed">
Line 115: Line 122:
 
<div id="BlackHoleExtra1"></div>
 
<div id="BlackHoleExtra1"></div>
  
=== Problem 5. ===
+
=== Problem 5: Schwarzschild is a vacuum solution ===
 
Show that Schwarzschild metric is a solution of Einstein's equation in vacuum.
 
Show that Schwarzschild metric is a solution of Einstein's equation in vacuum.
 
<div class="NavFrame collapsed">
 
<div class="NavFrame collapsed">
Line 128: Line 135:
 
For background on Killing vectors see problems [[Equations of General Relativity#equ_oto-kill1|K1]], [[Equations of General Relativity#equ_oto-kill2|K2]], [[Equations of General Relativity#equ_oto-kill3|K3]] of chapter 2.
 
For background on Killing vectors see problems [[Equations of General Relativity#equ_oto-kill1|K1]], [[Equations of General Relativity#equ_oto-kill2|K2]], [[Equations of General Relativity#equ_oto-kill3|K3]] of chapter 2.
 
<div id="BlackHole19"></div>
 
<div id="BlackHole19"></div>
=== Problem 6. ===
+
=== Problem 6: timelike Killing vector ===
What integral of motion arises due to existance of a timelike Killing vector? Express it through the physical velocity of the particle.
+
What integral of motion arises due to existence of a timelike Killing vector? Express it through the physical velocity of the particle.
 
<div class="NavFrame collapsed">
 
<div class="NavFrame collapsed">
 
   <div class="NavHead">solution</div>
 
   <div class="NavHead">solution</div>
 
   <div style="width:100%;" class="NavContent">
 
   <div style="width:100%;" class="NavContent">
 
     <p style="text-align: left;">
 
     <p style="text-align: left;">
If a Killing vector field is timelike we can use the coordinate frame in which $K^\mu$ is the unitary vector of the time coordinate\footnote{See problem \ref{equ_oto-kill2} of chapter 2.} $K^{\mu}=(1,0,0,0)$, while the spacelike basis vectors are orthogonal to it. Then the integral of motion is energy in the corresponding (stationary) frame $p_{\mu}K^{\mu}=p_{0}\equiv\varepsilon/c$.
+
If a Killing vector field is timelike we can use the coordinate frame in which $K^\mu$ is the unitary vector of the time coordinate (see problem [[Equations of General Relativity#equ_oto-kill2|K2]] of chapter 2.) $K^{\mu}=(1,0,0,0)$, while the spacelike basis vectors are orthogonal to it. Then the integral of motion is energy in the corresponding (stationary) frame $p_{\mu}K^{\mu}=p_{0}\equiv\varepsilon/c$.
  
Using the result (\ref{u0}), which holds for arbitrary gravitational field, we obtain the expression for energy, which is the integral of motion in a stationary metric
+
Using the result [[Technical_warm-up#u0|u0]], which holds for arbitrary gravitational field, we obtain the expression for energy, which is the integral of motion in a stationary metric
 
\begin{equation}\label{EnergyStat}
 
\begin{equation}\label{EnergyStat}
 
\varepsilon=mc^{2}u_{0}=
 
\varepsilon=mc^{2}u_{0}=
Line 144: Line 151:
 
   </div>
 
   </div>
 
</div>
 
</div>
 +
 +
  
 
<div id="BlackHole20"></div>
 
<div id="BlackHole20"></div>
  
=== Problem 7. ===
+
=== Problem 7: Killing vectors of a sphere ===
 
Derive the Killing vectors for a sphere in Cartesian coordinate system; in spherical coordinates.
 
Derive the Killing vectors for a sphere in Cartesian coordinate system; in spherical coordinates.
 
<div class="NavFrame collapsed">
 
<div class="NavFrame collapsed">
Line 179: Line 188:
 
<div id="BlackHole21"></div>
 
<div id="BlackHole21"></div>
  
=== Problem 8. ===
+
=== Problem 8: spherical symmetry of Schwarzshild ===
 
Verify that in coordinates $(t,r,\theta,\varphi)$ vectors  
 
Verify that in coordinates $(t,r,\theta,\varphi)$ vectors  
 
\[ \begin{array}{l}
 
\[ \begin{array}{l}
Line 197: Line 206:
  
 
<div id="BlackHole22"></div>
 
<div id="BlackHole22"></div>
 
+
=== Problem 9: planar motion ===
=== Problem 9. ===
+
 
Show that existence of Killing vectors $S^\mu$ and $T^\mu$ leads to motion of particles in a plane.
 
Show that existence of Killing vectors $S^\mu$ and $T^\mu$ leads to motion of particles in a plane.
 
<div class="NavFrame collapsed">
 
<div class="NavFrame collapsed">
Line 231: Line 239:
 
<div id="BlackHole23"></div>
 
<div id="BlackHole23"></div>
  
=== Problem 10. ===
+
=== Problem 10: stability of planar motion ===
 
Show that the particles' motion in the plane is stable.
 
Show that the particles' motion in the plane is stable.
 
<div class="NavFrame collapsed">
 
<div class="NavFrame collapsed">
Line 256: Line 264:
 
<div id="BlackHole24"></div>
 
<div id="BlackHole24"></div>
  
=== Problem 11. ===
+
=== Problem 11: remaining integrals of motion ===
 
Write down explicitly the conserved quantities  $p_{\mu}\Omega^{\mu}$ and $p_{\mu}R^{\mu}$ for movement in the plane $\theta=\pi/2$.
 
Write down explicitly the conserved quantities  $p_{\mu}\Omega^{\mu}$ and $p_{\mu}R^{\mu}$ for movement in the plane $\theta=\pi/2$.
 
<div class="NavFrame collapsed">
 
<div class="NavFrame collapsed">
Line 262: Line 270:
 
   <div style="width:100%;" class="NavContent">
 
   <div style="width:100%;" class="NavContent">
 
     <p style="text-align: left;">
 
     <p style="text-align: left;">
The first integral of motion is, up to a multiplier, the energy (see \ref{EnergyStat}):
+
The first integral of motion is, up to a multiplier, the energy (see Eq. [[Technical_warm-up#EnergyStat|EnergyStat]]):
 
\begin{equation}\label{SchwInt-E1}
 
\begin{equation}\label{SchwInt-E1}
 
E=\Omega_{\mu}u^{\mu}=
 
E=\Omega_{\mu}u^{\mu}=
 
g_{00}\frac{dx^{0}}{d\lambda}=h\frac{dt}{d\lambda}=
 
g_{00}\frac{dx^{0}}{d\lambda}=h\frac{dt}{d\lambda}=
 
\left(1-\frac{r_g}{r}\right)\frac{dt}{d\lambda},\end{equation}
 
\left(1-\frac{r_g}{r}\right)\frac{dt}{d\lambda},\end{equation}
where $\lambda$ is the geodesic parameter: for a massive particle we can always choose the natural parametrization  $d\lambda=ds=\gamma^{-1}\sqrt{g_{00}}\,dt$ (see (\ref{IntervalStaticCase})) so that $E$ is energy per unit mass; recovering the multipliers, for true energy we obtain
+
where $\lambda$ is the geodesic parameter: for a massive particle we can always choose the natural parametrization  $d\lambda=ds=\gamma^{-1}\sqrt{g_{00}}\,dt$ (see Eq. [[Technical_warm-up#IntervalStaticCase|IntervalStaticCase]]) so that $E$ is energy per unit mass; recovering the multipliers, for true energy we obtain
 
\begin{equation}\label{SchwInt-E2}
 
\begin{equation}\label{SchwInt-E2}
 
\varepsilon_{m}\equiv mcE=
 
\varepsilon_{m}\equiv mcE=
Line 292: Line 300:
 
<div id="BlackHole25"></div>
 
<div id="BlackHole25"></div>
  
=== Problem 12. ===
+
=== Problem 12: work and mass ===
 
What is the work needed to pull a particle from the horizon to infinity? Will a black hole's mass change if we drop a particle with zero initial velocity from immediate proximity of the horizon?
 
What is the work needed to pull a particle from the horizon to infinity? Will a black hole's mass change if we drop a particle with zero initial velocity from immediate proximity of the horizon?
 
<div class="NavFrame collapsed">
 
<div class="NavFrame collapsed">
Line 311: Line 319:
  
 
<div id="BlackHole26"></div>
 
<div id="BlackHole26"></div>
=== Problem 13. ===
+
=== Problem 13: null geodesics ===
 
Derive the equation for null geodesics (worldlines of massless particles).
 
Derive the equation for null geodesics (worldlines of massless particles).
 
<div class="NavFrame collapsed">
 
<div class="NavFrame collapsed">
Line 329: Line 337:
  
 
<div id="BlackHole27"></div>
 
<div id="BlackHole27"></div>
 
+
=== Problem 14: geodesic motion of massive particle ===
=== Problem 14. ===
+
 
Use energy conservation to derive $v(r)$, $\dot{r}(r)=dr/dt$, $r(t)$ for a massive particle. Initial conditions: $g_{00}|_{\dot{r}=0}=h_{0}$ (the limiting case $h_{0}\to 1$ is especially interesting and simple).
 
Use energy conservation to derive $v(r)$, $\dot{r}(r)=dr/dt$, $r(t)$ for a massive particle. Initial conditions: $g_{00}|_{\dot{r}=0}=h_{0}$ (the limiting case $h_{0}\to 1$ is especially interesting and simple).
 
<div class="NavFrame collapsed">
 
<div class="NavFrame collapsed">
Line 386: Line 393:
  
 
<div id="BlackHole28"></div>
 
<div id="BlackHole28"></div>
 
+
=== Problem 15: radial motion in terms of proper time ===
=== Problem 15. ===
+
 
Show that the equation of radial motion in terms of proper time of the particle is the same as in the non-relativistic Newtonian theory. Calculate the proper time of the fall from $r=r_0$ to the center. Derive the first correction in $r_{g}/r$ to the Newtonian result. Initial velocity is zero.
 
Show that the equation of radial motion in terms of proper time of the particle is the same as in the non-relativistic Newtonian theory. Calculate the proper time of the fall from $r=r_0$ to the center. Derive the first correction in $r_{g}/r$ to the Newtonian result. Initial velocity is zero.
 
<div class="NavFrame collapsed">
 
<div class="NavFrame collapsed">
Line 448: Line 454:
 
<div id="BlackHole29"></div>
 
<div id="BlackHole29"></div>
  
=== Problem 16. ===
+
=== Problem 16: ultra-relativistic limit ===
 
Derive the equations of radial motion in the ultra-relativistic limit.
 
Derive the equations of radial motion in the ultra-relativistic limit.
 
<div class="NavFrame collapsed">
 
<div class="NavFrame collapsed">
Line 463: Line 469:
 
<div id="BlackHole30"></div>
 
<div id="BlackHole30"></div>
  
=== Problem 17. ===
+
=== Problem 17: communication from near the black hole ===
 
A particle (observer) falling into a black hole is emitting photons, which are detected on the same radial line far away from the horizon (i.e. the photons travel from emitter to detector radially). Find  $r$, $v$ and $\dot{r}$ as functions of the signal's detection time in the limit  $r\to r_g$.
 
A particle (observer) falling into a black hole is emitting photons, which are detected on the same radial line far away from the horizon (i.e. the photons travel from emitter to detector radially). Find  $r$, $v$ and $\dot{r}$ as functions of the signal's detection time in the limit  $r\to r_g$.
 
<div class="NavFrame collapsed">
 
<div class="NavFrame collapsed">
Line 500: Line 506:
  
 
<div id="BlackHole31"></div>
 
<div id="BlackHole31"></div>
=== Problem 18. ===
+
=== Problem 18: stationary source and detector ===
 
The source and detector are stationary.
 
The source and detector are stationary.
 
<div class="NavFrame collapsed">
 
<div class="NavFrame collapsed">
Line 533: Line 539:
  
 
<div id="BlackHole32"></div>
 
<div id="BlackHole32"></div>
 
+
=== Problem 19: free-falling source ===
=== Problem 19. ===
+
The source is falling freely without initial velocity from radius $r_0$; it flies by the stationary detector at the moment of emission.
problem formulation
+
 
<div class="NavFrame collapsed">
 
<div class="NavFrame collapsed">
 
   <div class="NavHead">solution</div>
 
   <div class="NavHead">solution</div>
 
   <div style="width:100%;" class="NavContent">
 
   <div style="width:100%;" class="NavContent">
 
     <p style="text-align: left;">
 
     <p style="text-align: left;">
problem solution </p>
+
It is educative to derive the Doppler effect from analogous considerations. Let us consider an emitter (detector) moving radially with physical velocity $v$ (see. (\ref{Schw-v(u)})). From the normalizing condition
 +
\begin{equation}\label{Schw-rad-u0}
 +
1=u^{\mu}u_{\mu}=g_{00}(u^{0})^{2}+g_{11}(u^{1})^{2}=
 +
g_{00}(u^0)^{2}(1-\beta^{2})=\gamma^{-2} h (u^0)^{2},\end{equation}
 +
thus
 +
\begin{equation}\label{Schw-rad-u^mu}
 +
u^{\mu}=\gamma\Big(
 +
\frac{1}{\sqrt{g_{00}}},
 +
\frac{\beta}{\sqrt{-g_{11}}},0,0\Big)=
 +
\gamma\Big(\frac{1}{\sqrt{h}},
 +
\beta\sqrt{h},0,0\Big).\end{equation}
 +
For light
 +
\[0=k^{\mu}k_{\mu}=g_{00}(k^0)^{2}+g_{11}(k^{1})^{2}
 +
\quad\Rightarrow\quad
 +
k^{1}=\pm k^{0}\sqrt{-g_{00}/g_{11}}=\pm hk^{0},\]
 +
therefor the observer registers (or the detector emits) light with frequency
 +
\[\omega_{em}\equiv\omega_{\beta}=
 +
g_{\mu}u^{\mu}k^{\nu}=
 +
g_{00}u^{0}k^{0}+g_{11}u^{1}k^{1}=
 +
h\cdot \frac{\gamma}{\sqrt{h}}k^{0}\pm
 +
\frac{1}{h}\cdot\gamma\beta\sqrt{h}\cdot hk^{0}=
 +
\sqrt{h}\gamma k^{0}(1\pm\beta).\]
 +
Choosing the sign "$+$" here, which corresponds to the emitter moving towards the horizon and light travaling outwards, we obtain the relativistic Dopper effect
 +
\begin{equation}\label{RelatDoppler-radial}
 +
\omega_{\beta}=
 +
\frac{\omega_{0}}{\sqrt{h}}\cdot\gamma(1+\beta)=
 +
\omega_{stat}\cdot\gamma(1+\beta).\end{equation}
 +
Taking into account that in this case the emitter is moving and detector is at rest, so that $\omega_{em}=\omega_{\beta}$, $\omega_{det}=\omega_{stat}$, we get
 +
\begin{equation}\label{Schw-RadialDoppler}
 +
\omega_{det}\equiv \omega_{stat}=
 +
\omega_{em}\frac{\gamma^{-1}}{1+\beta}=
 +
\omega_{em}\sqrt{\frac{1-\beta}{1+\beta}}.\end{equation} </p>
 
   </div>
 
   </div>
 
</div>
 
</div>
  
 
<div id="BlackHole33"></div>
 
<div id="BlackHole33"></div>
=== Problem 20. ===
+
 
problem formulation
+
=== Problem 20: adding the two effects ===
 +
The source is freely falling the same way, while the detector is stationary at $r_{det}>r_{em}$.
 
<div class="NavFrame collapsed">
 
<div class="NavFrame collapsed">
 
   <div class="NavHead">solution</div>
 
   <div class="NavHead">solution</div>
 
   <div style="width:100%;" class="NavContent">
 
   <div style="width:100%;" class="NavContent">
 
     <p style="text-align: left;">
 
     <p style="text-align: left;">
problem solution </p>
+
In this case we just need to take into account both the gravitational redshift (\ref{Schw-RedShift}) and the Doppler effect (\ref{Schw-RadialDoppler}). On substituting $\gamma=\sqrt{h_{0}/h_{em}}$, we get
 +
\begin{equation}\label{Schw-Rad-FullRedshift}
 +
\omega_{det}=\omega_{stat}(r_{em})
 +
\sqrt{\frac{h_{em}}{h_{det}}}=
 +
\omega_{em}\frac{\gamma^{-1}}{1+\beta}
 +
\sqrt{\frac{h_{em}}{h_{det}}}=
 +
\frac{\omega_{em}}{1+\beta}\cdot
 +
\frac{h_{em}}{\sqrt{h_{0}h_{det}}}\end{equation}
 +
In the limit $r_{em}\equiv r\sim r_{g}\ll r_{det}$ we obtain $\beta\approx 1$, thus
 +
\[\omega_{det}\approx
 +
\omega_{em}\cdot \frac{h_{em}}{2}=
 +
\omega_{em}\cdot {\textstyle\frac{1}{2}}
 +
\Big(1-\frac{r_{g}}{r_{em}}\Big)\to 0.\]</p>
 
   </div>
 
   </div>
 
</div>
 
</div>
  
 
<div id="BlackHole34"></div>
 
<div id="BlackHole34"></div>
=== Problem 21. ===
+
=== Problem 21: intensity ===
problem formulation
+
The source is falling freely and emitting continuously photons with constant frequency, the detector is stationary far away from the horizon $r_{det}\gg r_{g}$. How does the detected light's intensity depend on $r_{em}$ at the moment of emission? On the proper time of detector?
 
<div class="NavFrame collapsed">
 
<div class="NavFrame collapsed">
 
   <div class="NavHead">solution</div>
 
   <div class="NavHead">solution</div>
 
   <div style="width:100%;" class="NavContent">
 
   <div style="width:100%;" class="NavContent">
 
     <p style="text-align: left;">
 
     <p style="text-align: left;">
problem solution </p>
+
Repeating the argument of [[Schwarzschild black hole#BlackHole30|this problem]], for the proper time interval between detection of consecutive infinitely close signals (\ref{Schw-dtL(dr)}) we have
 +
\[dt_{\Lambda}=
 +
-\frac{dr}{h}\Big(\frac{1}{v}+\frac{r_{g}}{r}\Big),\]
 +
where $dr$ is the corresponding displacement of the emitter, related with its proper time interval as (see \ref{ShwRad-tau})
 +
\[dr=-\gamma v h\, d\tau.\]
 +
Then using $\gamma=h^{-1/2}$ from (\ref{SchwRad-V}), we get
 +
\begin{equation}\label{Schw-dtL(dTau)}
 +
dt_{\Lambda}=\gamma
 +
\Big(1+v\frac{r_{g}}{r}\Big) d\tau=
 +
h^{-1/2}\Big(1+v\frac{r_{g}}{r}\Big) d\tau.\end{equation}
 +
Intensity of the emitted and detected light is proportional to the photons' frequency and inversely proportional to the proper time interval during which a given number $N$ of photons are being emitted/detected. So, combining (\ref{Schw-Rad-FullRedshift}) and (\ref{Schw-dtL(dTau)}), we get
 +
\begin{equation}\label{SchwRad-I}
 +
\frac{I_{det}}{I_{em}}=
 +
\frac{\omega_{det}}{\omega_{em}}\cdot
 +
\frac{d\tau}{dt_{\Lambda}}=
 +
\frac{1}{1+v}
 +
\frac{h_{em}}{\sqrt{h_{0}h_{det}}}\cdot
 +
\frac{\sqrt{h_{em}}}{1+v\frac{r_{g}}{r}}.\end{equation}
 +
In the limit $h_{0},h_{det}\to 1$ and $r\to r_{g}$, so that $v\to1$, we have
 +
\[\frac{I_{det}}{I_{em}}\approx
 +
\frac{1}{4}h_{em}^{3/2}=
 +
\frac{1}{4}\Big(1-\frac{r_{g}}{r}\Big)^{3/2}.\]
 +
Substituting $r(t_\Lambda)$ from (\ref{Schw-r(tL)}), we finally obtain
 +
\begin{equation}\label{Schw-I(tL)}
 +
I_{det}(t_\Lambda)\approx I_{em}\cdot
 +
\exp\Big\{-\frac{3\;t_\Lambda}{4\;r_g}\Big\}.\end{equation} </p>
 
   </div>
 
   </div>
 
</div>
 
</div>
Line 570: Line 644:
  
 
<div id="BlackHole35"></div>
 
<div id="BlackHole35"></div>
=== Problem 22. ===
+
=== Problem 22: impact parameter ===
problem formulation
+
Show that the ratio of specific energy to specific angular momentum of a particle equals to $r_{g}/b$, where $b$ is the impact parameter at infinity (for unbounded motion).
 
<div class="NavFrame collapsed">
 
<div class="NavFrame collapsed">
 
   <div class="NavHead">solution</div>
 
   <div class="NavHead">solution</div>
 
   <div style="width:100%;" class="NavContent">
 
   <div style="width:100%;" class="NavContent">
 
     <p style="text-align: left;">
 
     <p style="text-align: left;">
problem solution </p>
+
For $r\gg r_{g}$ we'll have $h\approx 1$ so if we count the angle from $\pi/2$ the impact parameter is $b=r\cos\varphi$. If we denote the length element of the trajectory by $dl$, then the ratio between the angular momentum and energy is
 +
\[\frac{L}{E}
 +
=\frac{r^2\cdot d\varphi/d\lambda}
 +
{h\cdot dt/d\lambda}
 +
=\frac{r^2 d\varphi}{hdt}\approx
 +
\frac{r^2 (dl\cos\varphi\, /r)}{dl/v}=
 +
r\cdot r\cos\varphi=vb.\]
 +
In the ultrarelativistic case
 +
\[L=Eb.\]</p>
 
   </div>
 
   </div>
 
</div>
 
</div>
  
 
<div id="BlackHole36"></div>
 
<div id="BlackHole36"></div>
=== Problem 23. ===
+
 
problem formulation
+
=== Problem 23: geodesic equations and effective potential ===
 +
Derive the geodesics' equations; bring the equation for $r(\lambda)$ to the form
 +
\[\frac{1}{2}\Big(\frac{dr}{d\lambda}\Big)^{2}
 +
+V_{\epsilon}(r)=\varepsilon,\]
 +
where $V_{\epsilon}(r)$ is a function conventionally termed as effective potential.
 
<div class="NavFrame collapsed">
 
<div class="NavFrame collapsed">
 
   <div class="NavHead">solution</div>
 
   <div class="NavHead">solution</div>
 
   <div style="width:100%;" class="NavContent">
 
   <div style="width:100%;" class="NavContent">
 
     <p style="text-align: left;">
 
     <p style="text-align: left;">
problem solution </p>
+
The first equation is $\epsilon=u^{\mu}u_{\mu}$. For a massive particle $\epsilon=1$ and we can also choose the parameter $\lambda$ as the natural parameter of the geodesic $d\lambda=ds$. For a massless particle $\epsilon=0$ and the parameter is arbitrary. In the Schwarzschild metric therefore
 +
\begin{equation}\label{Schw-Orbit1}
 +
h\Big(\frac{dt}{d\lambda}\Big)^{2}-
 +
\frac{1}{h}\Big(\frac{dr}{d\lambda}\Big)^{2}-
 +
r^{2}\Big(\frac{d\varphi}{d\lambda}\Big)^{2}=
 +
\epsilon.\end{equation}
 +
The two integrals of motion (\ref{SchwInt-E1},\ref{SchwInt-L1}) are
 +
\begin{equation}\label{Schw-Orbit-Integrals}
 +
E=h\frac{dt}{d\lambda};\qquad
 +
L=r^{2}\frac{d\varphi}{d\lambda}.\end{equation}
 +
Substituting this into (\ref{Schw-Orbit1}) and rearranging terms, we derive the equation for $r(\lambda)$ in the form
 +
\begin{equation}\label{Schw-V}
 +
\frac{1}{2}\Big(\frac{dr}{d\lambda}\Big)^{2}+
 +
V_{\epsilon}(r)=\varepsilon,\qquad\mbox{where}\quad
 +
V(r)=\frac{h}{2}
 +
\Big(\epsilon+\frac{L^2}{r^2}\Big),
 +
\quad \varepsilon=\frac{E^2}{2}.\end{equation}
 +
This is an analogue of one-dimensional motion of a particle in a potential $V$, with the quantity $\varepsilon$ playing the role of full energy (its analytic solution for the given $V$ is expressed through the integral from the square root of a third degree polynomial in the denominator, which is reduced to elliptic Jacobi functions). Note that $V(r=r_{g})=0$.
 +
 
 +
Recall that for massive particles $E$ and $L$ with the chosen parametrization are the energy and angular momentum per unit mass (see (\ref{SchwInt-E2},\ref{SchwInt-L2})).</p>
 
   </div>
 
   </div>
 
</div>
 
</div>
  
 
<div id="BlackHole37"></div>
 
<div id="BlackHole37"></div>
=== Problem 24. ===
+
 
problem formulation
+
=== Problem 24: bound and unbound motion ===
 +
Plot and investigate the function $V(r)$. Find the radii of circular orbits and analyze their stability; find the regions of parameters $(E,L)$ corresponding to bound and unbound motion, fall into the black hole. Consider the cases of a) massless, b) massive particles.
 
<div class="NavFrame collapsed">
 
<div class="NavFrame collapsed">
 
   <div class="NavHead">solution</div>
 
   <div class="NavHead">solution</div>
 
   <div style="width:100%;" class="NavContent">
 
   <div style="width:100%;" class="NavContent">
 
     <p style="text-align: left;">
 
     <p style="text-align: left;">
problem solution </p>
+
Let us first consider $\epsilon=0$.</p>
 +
 
 +
[[File:BHfig-SchwPhaL.png|center|thumb|400px|The effective potential for a massless particle $V(r/r_{g})$. The position of the maximum, that corresponds to the photon sphere, is at $r/r_{g}=\tfrac{3}{2}$ and does not depend on $L$]]
 +
 
 +
<p style="text-align: left;"> For massless particles the parameter $\lambda$ is arbitrary, so we will fix it by demanding that far from the horizon, where the metric is asymptotically Euclidean, holds $u^{0}\equiv dt/d\lambda=1$. Then, first, $E=1$, and second, at $r\gg r_{g}$
 +
\[0=u^{\mu}u_{\mu}=(u^{0})^{2}-u^{\alpha}u_{\alpha}
 +
\quad\Rightarrow\quad
 +
\frac{dl}{d\lambda}=1.\]
 +
Here $dl$ is the line element of the (asymptotically) flat space, and for such parametrization we obtain that $L=b$, where $b$ is the impact parameter of the ray at infinity:
 +
\begin{equation}\label{Schw-orb-NullInts}
 +
u^{0}|_{r\to\infty}=1,\;\Rightarrow\quad
 +
E=1;\quad L=b.\end{equation}
 +
Then
 +
\[V_{\epsilon=0}=b^{2}\frac{h}{2r^{2}}=
 +
\frac{b^{2}}{2}
 +
\Big(\frac{1}{r^2}-\frac{r_{g}}{r^3}\Big),
 +
\quad \varepsilon=\frac{1}{2}.\]
 +
At small $r/r_{g}$ the effective potential behaves as $\sim (-r^{-3})$, while at large $r/r_{g}$ as $\sim r^{2}$, and reaches its maximum at
 +
\[r_{max}=\frac{3}{2}r_{g},\quad
 +
V_{max}=V(r_{max})=
 +
\frac{2}{27}\Big(\frac{b}{r_g}\Big)^2.\]
 +
As this is a maximum, the corresponding circular orbit on the so-called "photon sphere" is unstable. For $\varepsilon>V_{max}$ all the trajectories from one side of it escape to infinity, and the ones from the other side fall on the center (more accurately, they fall on the horizon, as we do not yet consider the motion beyond this point). Rewriting the inequality in terms of $b$, we have
 +
\[b<b_{m}\equiv\frac{3\sqrt{3}}{2}\;r_{g}.\]
 +
At larger impact parameters there is a "turning point" for motion from infinity, in which $dr/d\lambda=0$. In the region $r<r_{\max}$ these values of $b$, which do not have the meaning of the impact parameter in this case, correspond to finite orbits falling on the center.</p>
 +
    <p style="text-align: left;">
 +
For massive particles it is convenient to express $V$ in terms of dimensionless quantities $\xi=r/r_{g}$ and $l=L/r_{g}$:
 +
\[V_{\epsilon=1}-\frac{1}{2}=-\frac{1}{2\xi}+
 +
\frac{l^2}{2\xi^2}-\frac{l^2}{2\xi^3}.\]
 +
The first term is the Newtonian potential energy, the second one is the centrifugal energy, and only the third term is absent in the Newtonian theory and is unique to General Relativity. It changes the asymptote of $V$ at small $\xi$: $V\sim -\xi^{-3}$ instead of the usual $V\sim\xi^{-2}$.</p>
 +
 
 +
[[File:BHfig-SchwM1aL.png|center|thumb|400px|Effective potential for a massive particle  $V(r/r_{g})$ for $l=\{0, 1.25, \sqrt{3}, 1.85, 2, 2.25, 2.5, 3\}\;\;\;$. There are stable as well as unstable circular orbits. The limiting value $l=\sqrt{3}$ defines the inflection point.]]
 +
[[File:BHfig-SchwM2aL.png|center|thumb|400px|Same figure zoomed in to show the shallow minima]]
 +
 
 +
<p style="text-align: left;"> The extrema of $V$ are found as the roots of quadratic equation
 +
\[\xi^2-2l^2 \xi +3l^2=0\quad\Rightarrow\quad
 +
\xi_{\pm}=l^{2}\left\{1\pm\sqrt{1-3/l^2}\right\}.\]
 +
Two extrema exist, maximum and minimum, if $l>\sqrt{3}$, i.e. $L>L_{cr}$, where
 +
\[L_{cr}\equiv\sqrt{12}\,GM.\]
 +
The minimum $\xi=\xi_{+}>l^2$ corresponds to a stable circular orbit and non-circular finite motions dangling around it (they are not elliptic). The maximum $\xi=\xi_{-}<l^2$ corresponds to an unstable circular orbit. If $\varepsilon>V(\xi_{-})$, then the motion is infinite with the fall on the center (i.e. at least on the horizon $r=r_g$).
 +
 
 +
The radius of the stable circular orbit is minimal when the discriminant turns to zero: $l=\sqrt{3}$,
 +
$\xi_{+}=\xi_{-}=l^{2}=3$, and thus \[r_{circ}^{min}=3r_{g}=6GM.\]
 +
 
 +
For $l<\sqrt{3}$, i.e. $L<L_{cr}$, there are no extrema of  $V$ and a particle's motion, either finite ($E<1$) or infinite ($E\geq 1$) always ends with the fall on the center.
 +
 
 +
In the limit $l\gg 1$, which corresponds to $L\gg r_{g}$,
 +
\[\xi_{-}\approx3/2,\;\xi_{+}\approx2l^2,
 +
\quad\Rightarrow\quad r_{-}=\frac{3}{2}r_{g};\;
 +
r_{+}=\frac{2L^{2}}{r_{g}}=
 +
\frac{L^2}{GM},\]
 +
so the inner unstable orbit tends to the photon sphere, while the outer stable orbit tends to the classical circular one.
 +
</p>
 
   </div>
 
   </div>
 
</div>
 
</div>
  
 
<div id="BlackHole38"></div>
 
<div id="BlackHole38"></div>
=== Problem 25. ===
+
 
problem formulation
+
=== Problem 25: gravitational cross-section ===
 +
Derive the gravitational capture cross-section for a massless particle; the first correction to it for a massive particle ultra-relativistic at infinity. Find the cross-section for a non-relativistic particle to the first order in $v^2/c^2$.
 
<div class="NavFrame collapsed">
 
<div class="NavFrame collapsed">
 
   <div class="NavHead">solution</div>
 
   <div class="NavHead">solution</div>
 
   <div style="width:100%;" class="NavContent">
 
   <div style="width:100%;" class="NavContent">
 
     <p style="text-align: left;">
 
     <p style="text-align: left;">
problem solution </p>
+
The gravitational capture happens, i.e. a particle moving from infinity falls on the center, if it passes above the effective potential barrier
 +
\[\varepsilon\geq V_{\max}.\]
 +
(a) For a massless particle we have already found the height of the barrier, so the limiting condition is
 +
\[\frac{1}{2}=\frac{2}{27}\Big(\frac{b}{r_g}\Big)^2,\]
 +
and for the capture cross-section we get
 +
\[S_{\gamma}=\pi b^2 =\frac{27\pi}{4}r_{g}^{2}
 +
=27\pi \Big(\frac{GM}{c^2}\Big)^2.\]
 +
 
 +
For a massive particle $V_{\epsilon=1}$ reaches its maximum in the lesser of the two roots $\xi_{\pm}$:
 +
\[\xi_{\max}^{-1}=\frac{1}{3}\big(1+\sqrt{1-3/l^2}\big).\]
 +
On substituting this into $V(\xi)$, after some transformations we find the maximum:
 +
\[V_{\max}=\frac{1}{27}\Big(
 +
l^2 +9+\frac{(l^2-3)^{3/2}}{l}\Big).\]
 +
 
 +
(b) Ultrarelativistic case:
 +
\[\gamma\gg1,\quad E=\gamma_{\infty}\gg1,
 +
\quad\varepsilon
 +
=\tfrac{1}{2}\gamma_{\infty}^{2}\gg 1,\]
 +
thus $V_{\max}\gg 1$, which only can be when $l\gg1$. In this limit, in the first order by $l^{-2}$
 +
\[V_{\max}=\frac{2}{27}l^2 +\frac{1}{6},\]
 +
so the limiting condition of capture is
 +
\[l^{2}=\frac{27}{4}\big(
 +
\gamma_{\infty}^{2}-\tfrac{1}{3}\big).\]
 +
Expressing it through the impact parameter $b=L/Ev$, we are led to
 +
\[S_{\gamma\gg1}
 +
=\pi b^2=\pi r_{g}^{2}\frac{l^2}{v^2 E^2}
 +
=\frac{27\pi}{4}r_{g}^{2}\Big(
 +
1+\frac{2}{3\gamma_{\infty}^{2}}\Big).\]
 +
 
 +
(c) Nonrelativistic case: $E\approx (1+v^2 /2)$, $\varepsilon\approx \tfrac{1}{2}(1+v^2)$, and the limiting condition of capture takes the form
 +
\[1+v^2 =\frac{1}{27}\Big(
 +
l^2 +9+\frac{(l^2-3)^{3/2}}{l}\Big).\]
 +
In the zeroth order by $v^2$ its solution is $l=2$ (note that at $l=\sqrt{3}$ there is no maximum of $V$), while in the first order
 +
\[l^2 =4(1+2v^2).\]
 +
In terms of impact parameter then the cross-section is
 +
\[S_{v\ll 1}=\pi r_{g}^{2}\frac{l^2}{v^2 E^2}
 +
\approx \frac{4\pi r_{g}^{2}}{v^2}(1+v^2).\]</p>
 
   </div>
 
   </div>
 
</div>
 
</div>
  
 
<div id="BlackHole39"></div>
 
<div id="BlackHole39"></div>
=== Problem 26. ===
+
=== Problem 26: innermost stable circular orbit ===
problem formulation
+
Find the minimal radius of stable circular orbit and its parameters. What is the maximum gravitational binding energy of a particle in the Schwarzschild spacetime?
 
<div class="NavFrame collapsed">
 
<div class="NavFrame collapsed">
 
   <div class="NavHead">solution</div>
 
   <div class="NavHead">solution</div>
 
   <div style="width:100%;" class="NavContent">
 
   <div style="width:100%;" class="NavContent">
 
     <p style="text-align: left;">
 
     <p style="text-align: left;">
problem solution </p>
+
The radius of stable circular orbit is minimal when
 +
\[\xi_{\min}=l_{min}^{2}=3+0.\]
 +
The value of effective potential at $\xi=\xi_{min}$ determines the particle's energy on the corresponding circular orbit (\ref{Schw-V}):
 +
\[\frac{E^2}{2}=\varepsilon=V|_{\epsilon=1}
 +
=\frac{1}{2}\Big(1-\frac{1}{\xi}\Big)
 +
\Big(1+\frac{l^{2}}{\xi^2}\Big)=
 +
\frac{1-\xi_{\min}^{-2}}{2},\]
 +
where in the last equality we plugged in the value $\xi=\xi_{min}=l_{min}^{2}$. Thus
 +
\[E=\sqrt{1-\xi_{min}^{-2}}=\sqrt{\frac{8}{9}}
 +
=\frac{2\sqrt{2}}{3}.\]
 +
The binding energy then is (in the units of $mc^{2}$)
 +
\[1-E=1-\frac{2\sqrt{2}}{3}\approx 0,06.\] </p>
 
   </div>
 
   </div>
 
</div>
 
</div>
Line 627: Line 832:
  
 
<div id="BlackHole40"></div>
 
<div id="BlackHole40"></div>
=== Problem 27. ===
+
=== Problem 27: gravitational lensing ===
problem formulation
+
Gravitational lensing is the effect of deflection of a light beam's (photon's) trajectory in the gravitational field. Derive the deflection of a photon's trajectory in Schwarzschild metric in the limit $L/r_{g}\gg 1$. Show that it is twice the value for a massive particle with velocity close to $c$ in the Newtonian theory.
 +
 
 
<div class="NavFrame collapsed">
 
<div class="NavFrame collapsed">
 
   <div class="NavHead">solution</div>
 
   <div class="NavHead">solution</div>
 
   <div style="width:100%;" class="NavContent">
 
   <div style="width:100%;" class="NavContent">
 
     <p style="text-align: left;">
 
     <p style="text-align: left;">
problem solution </p>
+
We choose $\lambda$ for massless particles as before and obtain
 +
\[\Big(\frac{dr}{d\lambda}\Big)^{2}+
 +
b^{2}\frac{h}{r^2}=1;\qquad
 +
\frac{d\varphi}{d\lambda}=b\frac{1}{r^2}.\]
 +
Excluding $\lambda$, after integration we obtain $\varphi(r)$. We are interested here in the variation of angle when $r$ goes from infinity to the minimal value, fro which the square root is zero. In the Newtonian theory it is $\pi/2$, so looking for the first correction to this value, we write
 +
\[\Delta\varphi\big|_{\pi/2}=
 +
b\int\limits_{r_{min}}^{\infty}
 +
\frac{dr/r^2} {\sqrt{1-hb^{2}/r^2}}=
 +
\left\|
 +
x=\frac{b}{r},\;\varepsilon=\frac{r_{g}}{r}\ll 1
 +
\right\|=
 +
\int\limits_{0}^{x_{max}}
 +
\frac{dx}{\sqrt{1-x^2+\varepsilon x^3}}.\]
 +
 
 +
Changing variables as $x^{2}-\varepsilon x^3=y^2$ and using the small parameter $\varepsilon$, we can transform the integral to one that is easy to compute
 +
\[\Delta\varphi\big|_{\pi/2}\approx\int\limits_{0}^{1}
 +
\frac{dy}{\sqrt{1-y^2}}(1+\varepsilon y)=
 +
\frac{\pi}{2}+\varepsilon=
 +
\frac{\pi}{2}+\frac{r_{g}}{b}.\]
 +
The second term is the needed correction, which is half the full correction $\delta\phi$ to the angle's variation when a particle moves from infinity and back to infinity:
 +
\[\delta\varphi\big|_{\pi}=\frac{2r_{g}}{b}=
 +
\frac{4GM}{b c^2}.\]
 +
 
 +
Now let us calculate the same thing in the Newtonian theory for a fast particle. Integrals of motion are
 +
\begin{align*}
 +
&L=b v_{\infty}=r^{2}\dot{\varphi},\\
 +
&E=U+\frac{\dot{r}^2}{2}+\frac{(r\dot{\varphi})^2}{2}
 +
=\frac{\dot{r}^2}{2}-\frac{r_g}{2r}+\frac{L^2}{2r^2}.
 +
\end{align*}
 +
Then $\varphi(r)$ can be written as
 +
\[\varphi=L\int\frac{dr/r^2}
 +
{\sqrt{2E+\frac{r_g}{r}-\frac{L^2}{r^2}}},\]
 +
and changing variables to $u=r^{-1}$, we obtain for the variation of angle from infinity to the turning point
 +
\[\varphi|_{\pi/2}=\int\limits_{0}^{u_{max}}\frac{du}
 +
{\sqrt{b^{-2}-u^2+\frac{r_g}{L^2}u}}.\]
 +
The last term under the root is a small correction. Let us make another change of variables $u=u'+\varepsilon/2$, where $\varepsilon=r_{g}/L^2 \ll 1$. Then up to terms of higher order by $\varepsilon$ the integral is reduced to
 +
\[\varphi|_{\pi/2}=\int\limits_{-\varepsilon/2}^{b^{-1}}
 +
\frac{du'}{\sqrt{b^{-2}-{u'}^2}}=
 +
\int\limits_{-b\varepsilon/2}^{1}
 +
\frac{d\xi}{\sqrt{1-\xi^2}}
 +
=\frac{\pi}{2}+\frac{b\varepsilon}{2}.\]
 +
Then for motion from infinity to infinity the deflection of the trajectory from a straight line is
 +
\[\delta\varphi|_{\pi}=b\varepsilon=
 +
\frac{b r_g}{L^2}
 +
=\frac{r_g}{b}\frac{1}{v_{\infty}^2}
 +
\to \frac{r_g}{b}.\]
 +
This is exactly half the correct quantity given by GTR.</p>
 
   </div>
 
   </div>
 
</div>
 
</div>
  
 
<div id="BlackHole41"></div>
 
<div id="BlackHole41"></div>
=== Problem 28. ===
+
=== Problem 28: generalization of Newtonian potential ===
problem formulation
+
Show that the $4$-acceleration of a stationary particle in the Schwarzschild metric can be presented in the form
 +
\[a_{\mu}=-\partial_{\mu}\Phi,\quad
 +
\text{where}\quad \Phi=\ln \sqrt{g_{00}}
 +
=\tfrac{1}{2}\ln g_{00}\]
 +
is some generalization of the Newtonian gravitational potential.
 
<div class="NavFrame collapsed">
 
<div class="NavFrame collapsed">
 
   <div class="NavHead">solution</div>
 
   <div class="NavHead">solution</div>
 
   <div style="width:100%;" class="NavContent">
 
   <div style="width:100%;" class="NavContent">
 
     <p style="text-align: left;">
 
     <p style="text-align: left;">
problem solution </p>
+
For a static particle $u^{\mu}=(u^0,0,0,0)$, where $u^0$ is found from the normalizing condition
 +
\[1=u^\mu u_\mu = g_{00}(u^{0})^2\quad
 +
\Rightarrow\quad u^{0}=\frac{1}{\sqrt{g_{00}}}.\]
 +
Then $4$-acceleration is
 +
\[a^\mu=\frac{du^\mu}{ds}
 +
=-\Gamma^{\mu}_{\nu\lambda}u^{\nu}u^{\lambda}
 +
=-\Gamma^{\mu}_{00}(u^0)^{2}\]
 +
and
 +
\[a_{\mu}=-\frac{1}{g_{00}}\Gamma_{\mu\,00}
 +
=-\frac{1}{2g_{00}}(-\partial_{\mu}g_{00})
 +
=\frac{1}{2}\frac{\partial_{\mu}g_{00}}{g_{00}}
 +
=\partial_{\mu}\Phi,\]
 +
where $\Phi=\ln\sqrt{g_{00}}$. </p>
 
   </div>
 
   </div>
 
</div>
 
</div>
  
 
<div id="BlackHole42"></div>
 
<div id="BlackHole42"></div>
=== Problem 29. ===
+
=== Problem 29: coordinate-invariant reformulation ===
problem formulation
+
Let us reformulate the problem in a coordinate-independent manner. Suppose we have an arbitrary stationary metric with timelike Killing vector $\xi^\mu$, and we denote the $4$-velocity of a stationary observer by $u^{\mu}=\xi^{\mu}/V$. What is the $4$-force per unit mass that we need to apply to a test particle in order to make it stay stationary? Show in coordinate-independent way that the answer coincides with $\partial_{\mu}\Phi$ (up to the sign), and rewrite $\Phi$ in coordinate-independent form.
 
<div class="NavFrame collapsed">
 
<div class="NavFrame collapsed">
 
   <div class="NavHead">solution</div>
 
   <div class="NavHead">solution</div>
 
   <div style="width:100%;" class="NavContent">
 
   <div style="width:100%;" class="NavContent">
 
     <p style="text-align: left;">
 
     <p style="text-align: left;">
problem solution </p>
+
From the normalizing condition  $V^2=\xi^\mu \xi_\mu$. Here we will use a different meaning of acceleration, natural in the context of GTR. On a geodesic the covariant acceleration of a particle $w^\mu = u^\mu \nabla_{\mu} u^\nu$ is zero; if a particle moves not on a geodesic, this means that some (non-gravitational) force (per unit mass) is acting on it, equal to $w^{\mu}=-a^\mu$.
 +
 
 +
First of all, let us note that, as $\nabla g=0$,
 +
\[\xi^{\mu}\nabla_{\mu}V
 +
=\xi^{\mu}\xi^{\nu}
 +
(\nabla_{\mu}\xi_{\nu}+\nabla_{\nu}\xi_{\mu})=0,\]
 +
and also
 +
\[2V\nabla_{\mu}V=\nabla_{\mu}V^2
 +
=\nabla_{\mu}\xi^{\nu}\xi_{\nu}
 +
=2\xi^{\nu}\nabla_{\mu}\xi_{\nu}.\]
 +
Then using stationarity $u^\mu=\xi_\mu /V$ and the Killing equation, we get
 +
\[ -a^\mu = w^\mu=\frac{1}{V}\xi^{\nu}\nabla_{\nu}
 +
\big(\frac{1}{V}\xi^\mu\big)
 +
=\frac{1}{V^2}\xi_{\nu}\nabla^{\nu}\xi^{\mu}
 +
=-\frac{1}{V^2}\xi_{\nu}\nabla^{\mu}\xi^{\nu}
 +
=-\frac{1}{V}\nabla^{\mu}V=-\nabla^{\mu}\ln V,\]
 +
therefore
 +
\[a^{\mu}=\nabla^{\mu}\Phi,\quad\text{where}
 +
\quad \Phi=\ln V=\tfrac{1}{2}\ln \xi^{\mu}\xi_{\nu}.\]
 +
 
 +
In the weak field limit $g_{00}\approx 1+\tfrac{2\phi}{c^2}$, so $\Phi\approx \phi/c^2$. $\phi$ is the Newtonian gravitational potential. </p>
 
   </div>
 
   </div>
 
</div>
 
</div>
  
 
<div id="BlackHole43"></div>
 
<div id="BlackHole43"></div>
=== Problem 30. ===
+
=== Problem 30: surface gravity ===
problem formulation
+
Surface gravity $\kappa$ of the Schwarzschild horizon can be defined as acceleration of a stationary particle at the horizon, measured in the proper time of a stationary observer at infinity. Find $\kappa$.
 
<div class="NavFrame collapsed">
 
<div class="NavFrame collapsed">
 
   <div class="NavHead">solution</div>
 
   <div class="NavHead">solution</div>
 
   <div style="width:100%;" class="NavContent">
 
   <div style="width:100%;" class="NavContent">
 
     <p style="text-align: left;">
 
     <p style="text-align: left;">
problem solution </p>
+
The scalar acceleration near the horizon, measured in proper time, is
 +
\begin{align*}
 +
a=&\sqrt{|a^{\mu}a_{\mu}|}
 +
=\sqrt{|g^{\mu\nu}a_{\mu}a_{\nu}|}
 +
=\sqrt{|g^{11}|(a_1)^{1}}=\sqrt{|g^{11}|(\Phi')^2}=\\
 +
&=\sqrt{g_{00}}\;\Phi'(r)
 +
=\sqrt{g_{00}}\;\frac{g_{00}'}{g_{00}}
 +
=\frac{g_{00}'}{2\sqrt{g_{00}}}.
 +
\end{align*}
 +
It tends to infinity at the horizon. The time of remote observer $t$ is related to the proper time as $dt=d\tau\sqrt{g_{00}}$, therefore acceleration measured in time $t$ is
 +
\[a_{\infty}=\sqrt{g_{00}}\;a=\frac{1}{2}g_{00}',\]
 +
and it is finite. At $r=r_{g}$ it gives us the surface gravity
 +
\[\kappa=a_{\infty}\Big|_{r=r_g}=\frac{1}{2r_{g}}
 +
=\frac{c^4}{4MG}.\]
 +
In the last equality we restored the dimensional factor $c^2$. </p>
 
   </div>
 
   </div>
 
</div>
 
</div>
Line 676: Line 978:
  
 
<div id="BlackHole44"></div>
 
<div id="BlackHole44"></div>
=== Problem 31. ===
+
 
problem formulation
+
=== Problem 31: uniqueness in exterior region ===
 +
Suppose all the matter is distributed around the center of symmetry, and its energy-momentum tensor is spherically symmetric, so that the form of $g_{\mu\nu}$ written above is correct. Show that the solution in the exterior region is reduced to the Schwarzschild metric and find the relation between $C$ and the system's mass $M$.
 
<div class="NavFrame collapsed">
 
<div class="NavFrame collapsed">
 
   <div class="NavHead">solution</div>
 
   <div class="NavHead">solution</div>
 
   <div style="width:100%;" class="NavContent">
 
   <div style="width:100%;" class="NavContent">
 
     <p style="text-align: left;">
 
     <p style="text-align: left;">
problem solution </p>
+
We eliminate the factor $f$ in $g_{00}$ by coordinate transformation $\sqrt{f(t)}dt=dt'$; other components of the metric do not change. In the weak field limit  $g_{00}\approx(1+\tfrac{2\varphi}{c^2})$, where  $\varphi=-GM/r$ is the Newtonian gravitational potential. Comparing with the asymptote of $g_{00}$ we get
 +
\[C=r_{g}=\frac{2GM}{c^2}.\]</p>
 
   </div>
 
   </div>
 
</div>
 
</div>
  
 
<div id="BlackHole45"></div>
 
<div id="BlackHole45"></div>
=== Problem 32. ===
+
=== Problem 32: solution in a spherically symmetric void ===
problem formulation
+
Let there be a spherically symmetric void $r<r_{0}$ in the spherically symmetric matter distribution. Show that spacetime in the void is flat.
 
<div class="NavFrame collapsed">
 
<div class="NavFrame collapsed">
 
   <div class="NavHead">solution</div>
 
   <div class="NavHead">solution</div>
 
   <div style="width:100%;" class="NavContent">
 
   <div style="width:100%;" class="NavContent">
 
     <p style="text-align: left;">
 
     <p style="text-align: left;">
problem solution </p>
+
The integration constant is determined by the demand of boundedness of $g_{\mu\nu}$ in the region: $C=0$. Then on coordinate transformation $f(t)dt^2=d\tau^2$ we will obtain the flat Minkowskii metric
 +
\[ds^2=d\tau^2-dr^2-r^2 d\Omega^2.\] </p>
 
   </div>
 
   </div>
 
</div>
 
</div>
  
 
<div id="BlackHole46"></div>
 
<div id="BlackHole46"></div>
=== Problem 33. ===
+
=== Problem 33: shells ===
problem formulation
+
Let the matter distribution be spherically symmetric and filling regions  $r<r_{0}$ and $r_{1}<r<r_{2}$ ($r_{0}<r_{1}$). Can one affirm, that the solution in the layer of empty space $r_{0}<r<r_{1}$ is also the Schwarzschild metric?
 
<div class="NavFrame collapsed">
 
<div class="NavFrame collapsed">
 
   <div class="NavHead">solution</div>
 
   <div class="NavHead">solution</div>
 
   <div style="width:100%;" class="NavContent">
 
   <div style="width:100%;" class="NavContent">
 
     <p style="text-align: left;">
 
     <p style="text-align: left;">
problem solution </p>
+
With such matter distribution the solution in region $r>r_{2}$ is Schwarzschild, as the constant $C$ and function $f(t)$ are fixed in the usual way. In region $r_{0}<r<r_{1}$, however, we do not have the freedom to choose a new time coordinate, as it will be determined by the smooth sewing-up of the metric at the boundaries (in region $r_{1}<r<r_{2}$ the solution is quite different, of course). Therefore $f(t)\neq 1$ in the inner region. </p>
 +
  </div>
 +
</div>
 +
 
 +
<div id="BlackHoleEin1"></div>
 +
=== Problem 34: Einstein equations for spherically symmetric case ===
 +
Consider a static, spherically symmetric spacetime, described by metric
 +
\[ds^{2} =-f(r)dt^{2} +f^{-1} (r)dr^{2} +r^{2} d\Omega ^{2}, \]
 +
and write the Einstein's equations for it.
 +
<div class="NavFrame collapsed">
 +
  <div class="NavHead">solution</div>
 +
  <div style="width:100%;" class="NavContent">
 +
    <p style="text-align: left;">
 +
\[(-f)-rf'(r)=-8\pi GPr^{2},\]
 +
where $P=T_{r}^{r} $ is the radial pressure of the matter source. </p>
 
   </div>
 
   </div>
 
</div>
 
</div>
Line 712: Line 1,031:
  
 
<div id="BlackHole47"></div>
 
<div id="BlackHole47"></div>
=== Problem 34. ===
+
=== Problem 35: Rindler metric ===
problem formulation
+
Make coordinate transformation in the Schwarzschild metric near the horizon $(r-r_{g})\ll r_{g}$ by using physical distance to the horizon as a new radial coordinate instead of $r$, and show that in the new coordinates it reduces near the horizon to the Rindler metric.
 
<div class="NavFrame collapsed">
 
<div class="NavFrame collapsed">
 
   <div class="NavHead">solution</div>
 
   <div class="NavHead">solution</div>
 
   <div style="width:100%;" class="NavContent">
 
   <div style="width:100%;" class="NavContent">
 
     <p style="text-align: left;">
 
     <p style="text-align: left;">
problem solution </p>
+
The physical distance to the horizon along the "radius" $r$ at $(r-r_{g})\ll r_{g}$ is
 +
\[l=\int\limits_{r_{g}}^{r}\frac{dr}{\sqrt{1-r_{g}/r}}=
 +
\int\limits_{r_{g}}^{r}
 +
\frac{\sqrt{r}dr}{\sqrt{r-r_{g}}}\approx
 +
\sqrt{r_{g}}\int\limits_{0}^{r-r_{g}}
 +
\frac{d\xi}{\sqrt{\xi}}=2\sqrt{r_{g}(r-r_{g})}.\]
 +
Substituting this into the metric, we obtain (restoring the $c$ factors by dimensionality)
 +
\[ds^{2}=l^{2}d\omega^{2}-dl^{2}-r^{2}(l)d\Omega^{2},
 +
\qquad\mbox{where}\quad\omega=\frac{ct}{2r_{g}}.\]
 +
Comparing with the [[Technical_warm-up#Rindler|Rindler metric]], we see that $l\equiv\rho=\frac{c^2}{a}$, and $\tau=t\frac{l}{2r_g}$. Thus the static Schwarzschild metric near the horizon has the same form as Minkowskii metric for a uniformly accelerated observer with acceleration $a=\frac{c^2}{l}$, which tends to infinity at the horizon. Then using the inverse transformation, from Rindler to Minkowskii, we can turn the Schwarzschild metric near the horizon into the flat one, thus showing explicitly that $r=r_g$ is just a removable coordinate singularity.</p>
 
   </div>
 
   </div>
 
</div>
 
</div>
  
 
<div id="BlackHole48"></div>
 
<div id="BlackHole48"></div>
=== Problem 35. ===
+
 
problem formulation
+
=== Problem 36: tortoise coordinate ===
 +
Derive the Schwarzschild metric in coordinates $t$ and  $r^\star=r+r_{g}\ln|r-r_g|$. How do the null geodesics falling to the center look like in $(t,r^\star)$? What range of values of $r^\star$ corresponds to the region $r>r_g$?
 
<div class="NavFrame collapsed">
 
<div class="NavFrame collapsed">
 
   <div class="NavHead">solution</div>
 
   <div class="NavHead">solution</div>
 
   <div style="width:100%;" class="NavContent">
 
   <div style="width:100%;" class="NavContent">
 
     <p style="text-align: left;">
 
     <p style="text-align: left;">
problem solution </p>
+
Making the change of variables
 +
$(t,r)\to(t,r^\star)$, we get
 +
\[ds^2=\Big(1-\frac{r_g}{r}\Big)
 +
\big(dt^2-(dr^\star)^2\big)-r^2 d\Omega^2,\]
 +
where $r$ should be understood as a function of $r^\star$. This metric is conformally flat$^{*}$ (without its angular part), and with no singularities. This is achieved, however, by pulling the horizon to infinity: $r=r_g$ corresponds to $r^\star =-\infty$.</p>
 +
 
 +
<p style="text-align: left;">$^{*}$A metric $g_{\mu\nu}$ is said to be conformally flat if it differs from the Minkowskii metric $\eta_{\mu\nu}$ by a factor $g_{\mu\nu}=\omega^{2}(x)\eta_{\mu\nu}$.</p>
 
   </div>
 
   </div>
 
</div>
 
</div>
  
 
<div id="BlackHole49"></div>
 
<div id="BlackHole49"></div>
=== Problem 36. ===
+
 
problem formulation
+
=== Problem 37: introducing null coordinates ===
 +
Rewrite the metric in coordinates $r$ and $u=t-r^\star$, find the equations of null geodesics and the value of $g=det(g_{\mu\nu})$ at $r=r_{g}$. Likewise in coordinates $r$ and $v=t+r^\star$; in coordinates $(u,v)$. The coordinate frames $(v,r)$ and $(u,r)$ are called the ingoing and outgoing Eddington-Finkelstein coordinates.
 
<div class="NavFrame collapsed">
 
<div class="NavFrame collapsed">
 
   <div class="NavHead">solution</div>
 
   <div class="NavHead">solution</div>
 
   <div style="width:100%;" class="NavContent">
 
   <div style="width:100%;" class="NavContent">
 
     <p style="text-align: left;">
 
     <p style="text-align: left;">
problem solution </p>
+
In different coordinates
 +
\begin{align*}
 +
ds^2&=\Big(1-\frac{r_g}{r}\Big)dv^2
 +
-\big(dv\,dr+dr\,dv\big)-r^2 d\Omega^2=\\
 +
&=\Big(1-\frac{r_g}{r}\Big)du^2
 +
+\big(du\,dr+dr\,du\big)-r^2 d\Omega^2,\\
 +
&=\frac{1}{2}\Big(1-\frac{r_g}{r}\Big)
 +
\big(du\,dv+dv\,du\big)-r^2 d\Omega^2.
 +
\end{align*}
 +
The null geodesics equations are $v=const$ for the ones falling to the center and $u=const$ for the ones escaping to infinity. Thus, by using the null coordinates $(u,v)$, we rectify the geodesics everywhere including the vicinity of the horizon. In coordinates $(r,v)$ we can analytically extend the solution beyond the horizon. The resulting spacetime region contains all the geodesics corresponding to particles which cross the horizon from the outside in, as well as all the geodesics in the outer region.
 +
 
 +
In coordinates $(r,u)$ we can likewise extend the solution beyond the horizon, and the resulting spacetime will contain, in addition to all the geodesics in the outer region, all the geodesics corresponding to particles crossing the horizon from the inside out. The two considered analytic extensions extend out metric to "different sides". This will be seen better in Kruskal coordinates.
 +
 
 +
In coordinates $(r,u)$ and $(r,v)$ the determinant of $g_{\mu\nu}$ is $g=-r^{4}\sin^{2}\theta$, and in coordninates $(u,v)$ we get $g=-\frac{1}{4}(1-r_{g}/r)^{2}r^{4}\sin^{2}\theta$.</p>
 
   </div>
 
   </div>
 
</div>
 
</div>
  
 
<div id="BlackHole50"></div>
 
<div id="BlackHole50"></div>
=== Problem 37. ===
+
=== Problem 38: Kruskal-Sekeres metric ===
problem formulation
+
Rewrite the Schwarzschild metric in coordinates $(u',v')$ and in the Kruskal coordinates $(T,R)$ (Kruskal solution), defined as follows:
 +
\[v'=e^{v/2r_g},\quad u'=-e^{-u/2r_g};\qquad
 +
T=\frac{u'+v'}{2},\quad R=\frac{v'-u'}{2}.\]
 +
What are the equations of null geodesics, surfaces $r=const$ and $t=const$, of the horizon $r=r_{g}$, singularity $r=0$, in the coordinates $(T,R)$? What is the range space of $(T,R)$? Which regions in the Schwarzschild coordinates do the regions $\{\text{I}:\;R>|T|\}$, $\{\text{II}:\;T>|R|\}$, $\{\text{III}:\;R<-|T|\}$ and $\{\text{IV}:\;T<-|R|\}$ correspond to? Which of them are casually connected and which are not? What is the geometry of the spacelike slice $T=const$ and how does it evolve with time $T$?
 
<div class="NavFrame collapsed">
 
<div class="NavFrame collapsed">
 
   <div class="NavHead">solution</div>
 
   <div class="NavHead">solution</div>
 
   <div style="width:100%;" class="NavContent">
 
   <div style="width:100%;" class="NavContent">
 
     <p style="text-align: left;">
 
     <p style="text-align: left;">
problem solution </p>
+
After substitution we obtain
 +
\[ds^2=\frac{2r_{g}^3}{r}e^{-r/r_{g}}
 +
(dv'\,du'+du'\,dv')-r^{2}d\Omega^2
 +
=\frac{4r_{g}^3}{r}e^{-r/r_{g}}
 +
(dT^2-dR^2)-r^{2}d\Omega^2.\]
 +
Equations of null geodegics: $u=const$ and $v=const$, or in terms of $(T,R)$: $T=\pm R+const$. The surface  $r=const$ is mapped to a hyperbola $T^2-R^2=-2(r-r_g)e^{r/r_g}$; singularity to a hyperbola $T^2-R^2=2r_g$ with ''two'' branches also; the horizon to \emph{two} straight lines $T=\pm R$. A surface $t=const$ is mapped to straight line $T=R\tanh\frac{t}{r_g}$; at $t\to\pm\infty$ it coincides with the horizon. Every point $(T,R)$ here represents a two-sphere.</p>
 +
 
 +
[[File:BHfig-KruskalCE.png|center|thumb|400px|The Kuskal diagram in coordinates $(T,R)$ and the mapping onto it of the coordinate grid $(t,r)$. Singularity is two hyperbola branches, in the past and future, while the horizon is two straight lines $T=\pm R$. Null geodesics are also straight lines $T\pm R=const$]]
 +
 
 +
<p style="text-align: left;">The physical region of values $(R,T)$ is given by condition $r>0$ and the region between the two hyperbola branches $T=\pm\sqrt{2r_g+R^2}$, which represent the past and future singularities. Region I is the asymptotically flat space outside of the horizon (we are living here); region II is the analytical extension along timelike geodesics directed inside across the horizon (coordinates $(r,v)$ map regions I and II); region III is the analytic extension along the timelike geodesics directed away from the horizon (coordinates $(r,u)$ map regions I and III); region IV is \emph{another} asymptotically flat spacetime. It is not casually connected to our spacetime region I.
 +
 
 +
A three-dimensional slice $T=const$ of the entire spacetime for $|T|>\sqrt{2r_g}$ is comprised of two unconnected asymptotically flat regions; for $|T|<\sqrt{2r_g}$ there is a connection between them, the so-called wormhole, or the Einstein-Rosen bridge. This bridge, however, is spacelike: it is seen clearly from the Kruskal diagram that there are no timelike geodesics (or any other timelike curves either) that would correspond to particles coming from one asymptotically flat region into the other.</p>
 
   </div>
 
   </div>
 
</div>
 
</div>
  
 
<div id="BlackHole51"></div>
 
<div id="BlackHole51"></div>
=== Problem 38. ===
+
=== Problem 39: Penrose diagram ===
problem formulation
+
Pass to coordinates
 +
\[v''=\arctan\frac{v'}{\sqrt{r_g}},\quad
 +
u''=\arctan\frac{u'}{\sqrt{r_g}}\]
 +
and draw the spacetime diagram of the Kruskal solution in them.
 
<div class="NavFrame collapsed">
 
<div class="NavFrame collapsed">
 
   <div class="NavHead">solution</div>
 
   <div class="NavHead">solution</div>
 
   <div style="width:100%;" class="NavContent">
 
   <div style="width:100%;" class="NavContent">
 
     <p style="text-align: left;">
 
     <p style="text-align: left;">
problem solution </p>
+
The coordinates $(v'',u'')$ are needed in order to conformally map the entire Kruskal solution onto a bounded region of parameters $(v'',u'')$,
 +
\[|v''|<\frac{\pi}{2},\quad |u''|<\frac{\pi}{2},
 +
\quad |v''+u''|<\frac{\pi}{2}.\]
 +
This is a square with two opposite angles cut out, rotated by $\pi/4$. Conformality here means that null geodesics, and thus the casual structure, is left invariant under such transformation, and equations of the null geodesics are still $u''=\pm v'' +const$.
 +
[[File:BHfig-KruskalPenroseC2.png|center|thumb|400px|Penrose diagram for the Kruskal solution, with the grid of null geodesics]]</p>
 
   </div>
 
   </div>
 
</div>
 
</div>
  
 
<div id="BlackHole52"></div>
 
<div id="BlackHole52"></div>
=== Problem 39. ===
+
=== Problem 40: more realistic collapse ===
problem formulation
+
The Kruskal solution describes an eternal black hole. Suppose, for simplicity,  that some black hole is formed as a result of radial collapse of a spherically symmetric shell of massless particles. What part of the Kruskal solution will be realized, and what will not be? What is the casual structure of the resulting spacetime?
 
<div class="NavFrame collapsed">
 
<div class="NavFrame collapsed">
 
   <div class="NavHead">solution</div>
 
   <div class="NavHead">solution</div>
 
   <div style="width:100%;" class="NavContent">
 
   <div style="width:100%;" class="NavContent">
 
     <p style="text-align: left;">
 
     <p style="text-align: left;">
problem solution </p>
+
Outside of the collapsing shell the spacetime should be the Schwarzschild solution. As the particles are massless, their worldlines on the Kruskal diagram lie on the straight line $T-R=const$, and the exterior region contains a part of region I and a part of region II.</p>
 +
 
 +
[[File:BHfig-PenroseRealStarC.png|center|thumb|200px|Penrose diagram for the collapse of a massless spherically symmetric shell]]
 +
 
 +
<p style="text-align: left;">The inner region is just a chunk of Minkowskii spacetime, up to the moment of singularity formation. Sewing together of the tho solutions along the spheres of equal circumference $2\pi r$ gives us the full spacetime realized in the considered idealized case of collapse. We see that singularity and horizon are only in the future.</p>
 
   </div>
 
   </div>
 
</div>
 
</div>

Latest revision as of 10:02, 2 May 2013


The spherically symmetric solution of Einstein's equations in vacuum for the spacetime metric has the form$^{*}$ \begin{align}\label{Schw} ds^{2}=h(r)\,dt^2-h^{-1}(r)\,dr^2-r^2 d\Omega^{2}, &\qquad\mbox{where}\quad h(r)=1-\frac{r_g}{r};\quad r_{g}=\frac{2GM}{c^{2}};\\ d\Omega^{2}=d\theta^{2}+\sin^{2}\theta\, d\varphi^{2}&\;\text{is the metric of unit sphere.}\nonumber \end{align} The Birkhoff's theorem$^{**}$ (1923) states, that this solution is unique up to coordinate transformations. The quantity $r_g$ is called the Schwarzschild radius, or gravitational radius, $M$ is the mass of the central body or black hole.

$^{*}$ K. Schwarzschild, On the gravitational field of a mass point according to Einstein's theory, Sitzungsber. Preuss. Akad. Wiss. Phys. Math. Kl., p.189 (1916) (there's a translation of the original paper at arXiv:physics/9905030v1; please disregard the abstract/foreword, which is incorrect).

$^{**}$ G.D. Birkhoff, Relativity and Modern Physics, p.253, Harvard University Press, Cambridge (1923); J.T. Jebsen, "Ark. Mat. Ast. Fys." (Stockholm) 15, nr.18 (1921), see also arXiv:physics/0508163v2.

Contents

Simple problems

Problem 1: local time

Find the interval of local time (proper time of stationary observer) at a point $(r,\theta,\varphi)$ in terms of coordinate time $t$, and show that $t$ is the proper time of an observer at infinity. What happens when $r\to r_{g}$?

Problem 2: measuring distances

What is the physical distance between two points with coordinates $(r_{1},\theta,\varphi)$ and $(r_{2},\theta,\varphi)$? Between $(r,\theta,\varphi_{1})$ and $(r,\theta,\varphi_{2})$? How do these distances behave in the limit $r_{1},r\to r_{g}$?

Problem 3: the inner region

What would be the answers to the previous two questions for $r<r_g$ and why*? Why the Schwarzschild metric cannot be imagined as a system of "welded" rigid rods in $r<r_g$, as it can be in the external region?


Problem 4: acceleration

Calculate the acceleration of a test particle with zero velocity.

Problem 5: Schwarzschild is a vacuum solution

Show that Schwarzschild metric is a solution of Einstein's equation in vacuum.

Symmetries and integrals of motion

For background on Killing vectors see problems K1, K2, K3 of chapter 2.

Problem 6: timelike Killing vector

What integral of motion arises due to existence of a timelike Killing vector? Express it through the physical velocity of the particle.


Problem 7: Killing vectors of a sphere

Derive the Killing vectors for a sphere in Cartesian coordinate system; in spherical coordinates.

Problem 8: spherical symmetry of Schwarzshild

Verify that in coordinates $(t,r,\theta,\varphi)$ vectors \[ \begin{array}{l} \Omega^{\mu}=(1,0,0,0),\\ R^{\mu}=(0,0,0,1),\\ S^{\mu}=(0,0,\cos\varphi,-\cot\theta\sin\varphi),\\ T^{\mu}=(0,0,-\sin\varphi,-\cot\theta\cos\varphi) \end{array}\] are the Killing vectors of the Schwarzschild metric.

Problem 9: planar motion

Show that existence of Killing vectors $S^\mu$ and $T^\mu$ leads to motion of particles in a plane.

Problem 10: stability of planar motion

Show that the particles' motion in the plane is stable.

Problem 11: remaining integrals of motion

Write down explicitly the conserved quantities $p_{\mu}\Omega^{\mu}$ and $p_{\mu}R^{\mu}$ for movement in the plane $\theta=\pi/2$.

Problem 12: work and mass

What is the work needed to pull a particle from the horizon to infinity? Will a black hole's mass change if we drop a particle with zero initial velocity from immediate proximity of the horizon?

Radial motion

Consider a particle's radial motion: $\dot{\varphi}=\dot{\theta}=0$. In this problem one is especially interested in asymptotes of all functions as $r\to r_{g}$.

Let us set $c=1$ here and henceforth measure time in the units of length, so that $x^{0}=t$, $\beta=v$, etc., and introduce the notation \[h(r)\equiv g_{00}(r)=-\frac{1}{g_{11}(r)}= 1-\frac{r_{g}}{r} \underset{r\to r_g+0}{\longrightarrow}+0.\]

Problem 13: null geodesics

Derive the equation for null geodesics (worldlines of massless particles).

Problem 14: geodesic motion of massive particle

Use energy conservation to derive $v(r)$, $\dot{r}(r)=dr/dt$, $r(t)$ for a massive particle. Initial conditions: $g_{00}|_{\dot{r}=0}=h_{0}$ (the limiting case $h_{0}\to 1$ is especially interesting and simple).

Problem 15: radial motion in terms of proper time

Show that the equation of radial motion in terms of proper time of the particle is the same as in the non-relativistic Newtonian theory. Calculate the proper time of the fall from $r=r_0$ to the center. Derive the first correction in $r_{g}/r$ to the Newtonian result. Initial velocity is zero.

Problem 16: ultra-relativistic limit

Derive the equations of radial motion in the ultra-relativistic limit.

Problem 17: communication from near the black hole

A particle (observer) falling into a black hole is emitting photons, which are detected on the same radial line far away from the horizon (i.e. the photons travel from emitter to detector radially). Find $r$, $v$ and $\dot{r}$ as functions of the signal's detection time in the limit $r\to r_g$.

Blackness of black holes

A source radiates photons of frequency $\omega_i$, its radial coordinate at the time of emission is $r=r_{em}$. Find the frequency of photons registered by a detector situated at $r=r_{det}$ on the same radial line in different situations described below. By stationary observers here, we mean stationary in the static Schwarzschild metric; "radius" is the radial coordinate $r$.

Problem 18: stationary source and detector

The source and detector are stationary.

Problem 19: free-falling source

The source is falling freely without initial velocity from radius $r_0$; it flies by the stationary detector at the moment of emission.

Problem 20: adding the two effects

The source is freely falling the same way, while the detector is stationary at $r_{det}>r_{em}$.

Problem 21: intensity

The source is falling freely and emitting continuously photons with constant frequency, the detector is stationary far away from the horizon $r_{det}\gg r_{g}$. How does the detected light's intensity depend on $r_{em}$ at the moment of emission? On the proper time of detector?

Orbital motion, effective potential

Due to high symmetry of the Schwarzschild metric, a particle's worldline is completely determined by the normalizing condition $u^{\mu}u_{\mu}=\epsilon$, where $\epsilon=1$ for a massive particle and $\epsilon=0$ for a massless one, plus two conservation laws---of energy and angular momentum.

Problem 22: impact parameter

Show that the ratio of specific energy to specific angular momentum of a particle equals to $r_{g}/b$, where $b$ is the impact parameter at infinity (for unbounded motion).

Problem 23: geodesic equations and effective potential

Derive the geodesics' equations; bring the equation for $r(\lambda)$ to the form \[\frac{1}{2}\Big(\frac{dr}{d\lambda}\Big)^{2} +V_{\epsilon}(r)=\varepsilon,\] where $V_{\epsilon}(r)$ is a function conventionally termed as effective potential.

Problem 24: bound and unbound motion

Plot and investigate the function $V(r)$. Find the radii of circular orbits and analyze their stability; find the regions of parameters $(E,L)$ corresponding to bound and unbound motion, fall into the black hole. Consider the cases of a) massless, b) massive particles.

Problem 25: gravitational cross-section

Derive the gravitational capture cross-section for a massless particle; the first correction to it for a massive particle ultra-relativistic at infinity. Find the cross-section for a non-relativistic particle to the first order in $v^2/c^2$.

Problem 26: innermost stable circular orbit

Find the minimal radius of stable circular orbit and its parameters. What is the maximum gravitational binding energy of a particle in the Schwarzschild spacetime?

Miscellaneous problems

Problem 27: gravitational lensing

Gravitational lensing is the effect of deflection of a light beam's (photon's) trajectory in the gravitational field. Derive the deflection of a photon's trajectory in Schwarzschild metric in the limit $L/r_{g}\gg 1$. Show that it is twice the value for a massive particle with velocity close to $c$ in the Newtonian theory.

Problem 28: generalization of Newtonian potential

Show that the $4$-acceleration of a stationary particle in the Schwarzschild metric can be presented in the form \[a_{\mu}=-\partial_{\mu}\Phi,\quad \text{where}\quad \Phi=\ln \sqrt{g_{00}} =\tfrac{1}{2}\ln g_{00}\] is some generalization of the Newtonian gravitational potential.

Problem 29: coordinate-invariant reformulation

Let us reformulate the problem in a coordinate-independent manner. Suppose we have an arbitrary stationary metric with timelike Killing vector $\xi^\mu$, and we denote the $4$-velocity of a stationary observer by $u^{\mu}=\xi^{\mu}/V$. What is the $4$-force per unit mass that we need to apply to a test particle in order to make it stay stationary? Show in coordinate-independent way that the answer coincides with $\partial_{\mu}\Phi$ (up to the sign), and rewrite $\Phi$ in coordinate-independent form.

Problem 30: surface gravity

Surface gravity $\kappa$ of the Schwarzschild horizon can be defined as acceleration of a stationary particle at the horizon, measured in the proper time of a stationary observer at infinity. Find $\kappa$.

Solving Einstein's equations for a spherically symmetric metric of general form in vacuum (energy-momentum tensor equals to zero), one can reduce the metric to \[ds^2=f(t)\Big(1-\frac{C}{r}\Big)dt^2 -\Big(1-\frac{C}{r}\Big)^{-1}dr^2-r^2 d\Omega^2,\] where $C$ is some integration constant, and $f(t)$ an arbitrary function of time $t$.

Problem 31: uniqueness in exterior region

Suppose all the matter is distributed around the center of symmetry, and its energy-momentum tensor is spherically symmetric, so that the form of $g_{\mu\nu}$ written above is correct. Show that the solution in the exterior region is reduced to the Schwarzschild metric and find the relation between $C$ and the system's mass $M$.

Problem 32: solution in a spherically symmetric void

Let there be a spherically symmetric void $r<r_{0}$ in the spherically symmetric matter distribution. Show that spacetime in the void is flat.

Problem 33: shells

Let the matter distribution be spherically symmetric and filling regions $r<r_{0}$ and $r_{1}<r<r_{2}$ ($r_{0}<r_{1}$). Can one affirm, that the solution in the layer of empty space $r_{0}<r<r_{1}$ is also the Schwarzschild metric?

Problem 34: Einstein equations for spherically symmetric case

Consider a static, spherically symmetric spacetime, described by metric \[ds^{2} =-f(r)dt^{2} +f^{-1} (r)dr^{2} +r^{2} d\Omega ^{2}, \] and write the Einstein's equations for it.

Different coordinates, maximal extension

We saw that a particle's proper time of reaching the singularity is finite. However, the Schwarzschild metric has a (removable) coordinate singularity at $r=r_{g}$. In order to eliminate it and analyze the casual structure of the full solution, it is convenient to use other coordinate frames. Everywhere below we transform the coordinates $r$ and $t$, while leaving the angular part unchanged.

Problem 35: Rindler metric

Make coordinate transformation in the Schwarzschild metric near the horizon $(r-r_{g})\ll r_{g}$ by using physical distance to the horizon as a new radial coordinate instead of $r$, and show that in the new coordinates it reduces near the horizon to the Rindler metric.

Problem 36: tortoise coordinate

Derive the Schwarzschild metric in coordinates $t$ and $r^\star=r+r_{g}\ln|r-r_g|$. How do the null geodesics falling to the center look like in $(t,r^\star)$? What range of values of $r^\star$ corresponds to the region $r>r_g$?

Problem 37: introducing null coordinates

Rewrite the metric in coordinates $r$ and $u=t-r^\star$, find the equations of null geodesics and the value of $g=det(g_{\mu\nu})$ at $r=r_{g}$. Likewise in coordinates $r$ and $v=t+r^\star$; in coordinates $(u,v)$. The coordinate frames $(v,r)$ and $(u,r)$ are called the ingoing and outgoing Eddington-Finkelstein coordinates.

Problem 38: Kruskal-Sekeres metric

Rewrite the Schwarzschild metric in coordinates $(u',v')$ and in the Kruskal coordinates $(T,R)$ (Kruskal solution), defined as follows: \[v'=e^{v/2r_g},\quad u'=-e^{-u/2r_g};\qquad T=\frac{u'+v'}{2},\quad R=\frac{v'-u'}{2}.\] What are the equations of null geodesics, surfaces $r=const$ and $t=const$, of the horizon $r=r_{g}$, singularity $r=0$, in the coordinates $(T,R)$? What is the range space of $(T,R)$? Which regions in the Schwarzschild coordinates do the regions $\{\text{I}:\;R>|T|\}$, $\{\text{II}:\;T>|R|\}$, $\{\text{III}:\;R<-|T|\}$ and $\{\text{IV}:\;T<-|R|\}$ correspond to? Which of them are casually connected and which are not? What is the geometry of the spacelike slice $T=const$ and how does it evolve with time $T$?

Problem 39: Penrose diagram

Pass to coordinates \[v''=\arctan\frac{v'}{\sqrt{r_g}},\quad u''=\arctan\frac{u'}{\sqrt{r_g}}\] and draw the spacetime diagram of the Kruskal solution in them.

Problem 40: more realistic collapse

The Kruskal solution describes an eternal black hole. Suppose, for simplicity, that some black hole is formed as a result of radial collapse of a spherically symmetric shell of massless particles. What part of the Kruskal solution will be realized, and what will not be? What is the casual structure of the resulting spacetime?